Difference between revisions of "2003 AMC 10B Problems/Problem 1"

(AMC 10 B 2003- Problem 1)
 
(Redirected page to 2003 AMC 12B Problems/Problem 1)
 
(5 intermediate revisions by 4 users not shown)
Line 1: Line 1:
1.  Which of the following is the same as
+
#REDIRECT [[2003 AMC 12B Problems/Problem 1]]
 
 
<math>\frac{2-4+6-8+10-12+14}{3-6+9-12+15-18+21}</math>?
 
 
 
<math>\textbf{(A)}\ -1 \qquad
 
\textbf{(B)}\ -\frac23 \qquad
 
\textbf{(C)}\ \frac23 \qquad
 
\textbf{(D)}\ 1 \qquad
 
\textbf{(E)}\ \frac{14}{3}</math>
 

Latest revision as of 00:02, 5 January 2014